Last visit was: 21 May 2024, 20:59 It is currently 21 May 2024, 20:59
Close
GMAT Club Daily Prep
Thank you for using the timer - this advanced tool can estimate your performance and suggest more practice questions. We have subscribed you to Daily Prep Questions via email.

Customized
for You

we will pick new questions that match your level based on your Timer History

Track
Your Progress

every week, we’ll send you an estimated GMAT score based on your performance

Practice
Pays

we will pick new questions that match your level based on your Timer History
Not interested in getting valuable practice questions and articles delivered to your email? No problem, unsubscribe here.
Close
Request Expert Reply
Confirm Cancel
SORT BY:
Date
Tags:
Difficulty: 655-705 Level,    Weaken,                            
Show Tags
Hide Tags
Manager
Manager
Joined: 12 Jul 2017
Posts: 199
Own Kudos [?]: 214 [1]
Given Kudos: 442
Location: India
Schools: ISB '21 (A)
GMAT 1: 570 Q43 V26
GMAT 2: 690 Q50 V32
GPA: 3.8
Send PM
Tutor
Joined: 16 Oct 2010
Posts: 14891
Own Kudos [?]: 65409 [3]
Given Kudos: 431
Location: Pune, India
Send PM
Senior Manager
Senior Manager
Joined: 31 Jan 2019
Posts: 368
Own Kudos [?]: 716 [0]
Given Kudos: 67
Location: Switzerland
Concentration: General Management
GPA: 3.9
Send PM
Manager
Manager
Joined: 18 Aug 2017
Posts: 117
Own Kudos [?]: 38 [0]
Given Kudos: 599
Send PM
Re: To prevent a newly built dam on the Chiff River from blocking the rout [#permalink]
Hi Experts,

I have doubt on the meaning " have been declining slightly over the last 20 years"
I understand that the interpretation of the phrase would be:
" Populations have been declining slightly every year for the last 20 years: that is, let's say the declining rate were 1% each year -- or it would be 2% ,3%,4% which are quite subjective but can impact the argument-- , the total declining rate would amount to about 20%,
which is quite large.

So, I think D could be the answer.

Please explain.

Thank you.
Veritas Prep Representative
Joined: 26 Jul 2010
Posts: 416
Own Kudos [?]: 2961 [1]
Given Kudos: 63
Send PM
Re: To prevent a newly built dam on the Chiff River from blocking the rout [#permalink]
1
Bookmarks
Expert Reply
Top Contributor
ballest127 wrote:
Hi Experts,

I have doubt on the meaning " have been declining slightly over the last 20 years"
I understand that the interpretation of the phrase would be:
" Populations have been declining slightly every year for the last 20 years: that is, let's say the declining rate were 1% each year -- or it would be 2% ,3%,4% which are quite subjective but can impact the argument-- , the total declining rate would amount to about 20%,
which is quite large.

So, I think D could be the answer.

Please explain.

Thank you.


Good question - what's tricky with your hypothetical though is that we already know that the decline is much greater than 20%. The stimulus tells us that before the dam there were several thousand of fish that passed through that part of the river each day, and now we're down to 300. So it's reasonable to say that we're looking at a 90+% decrease ("several thousand" has to be at least 3,000 and probably more), so "declined slightly" just can't be the explanation there.
Stacy Blackman Consulting Director of Test Prep
Joined: 21 Dec 2014
Affiliations: Stacy Blackman Consulting
Posts: 237
Own Kudos [?]: 393 [1]
Given Kudos: 165
Location: United States (DC)
GMAT 1: 790 Q51 V51
GRE 1: Q170 V170

GRE 2: Q170 V170
GPA: 3.11
WE:Education (Education)
Send PM
Re: To prevent a newly built dam on the Chiff River from blocking the rout [#permalink]
1
Bookmarks
VeritasPrepBrian wrote:
ballest127 wrote:
Hi Experts,

I have doubt on the meaning " have been declining slightly over the last 20 years"
I understand that the interpretation of the phrase would be:
" Populations have been declining slightly every year for the last 20 years: that is, let's say the declining rate were 1% each year -- or it would be 2% ,3%,4% which are quite subjective but can impact the argument-- , the total declining rate would amount to about 20%,
which is quite large.

So, I think D could be the answer.

Please explain.

Thank you.


Good question - what's tricky with your hypothetical though is that we already know that the decline is much greater than 20%. The stimulus tells us that before the dam there were several thousand of fish that passed through that part of the river each day, and now we're down to 300. So it's reasonable to say that we're looking at a 90+% decrease ("several thousand" has to be at least 3,000 and probably more), so "declined slightly" just can't be the explanation there.


I agree with Brian's comment.

But beyond that, note that the timeline here is a little funky for D. The passage says "before the construction of the dam and fish pass" but D says "over the last 20 years." Since we have no reason to believe that the dam took 20+ years to complete -- and we cannot simply assume that it did -- the story of the drop-off would need to be even more abrupt (and thus even less explainable through "declining slightly" over a 20-year span).

Oh, and I don't really buy that "declining slightly over the last 20 years" could mean "declining slightly each year, so that over the 20-year span it adds up to a really substantial, not slight, decline." That seems like it's just twisting the language way too hard.
Intern
Intern
Joined: 21 Apr 2018
Posts: 47
Own Kudos [?]: 10 [0]
Given Kudos: 14
Location: India
Schools: ISB '24 (A)
GMAT 1: 710 Q50 V36
GPA: 3.3
Send PM
Re: To prevent a newly built dam on the Chiff River from blocking the rout [#permalink]
I chose D, the reason being that since there has been a decline the population should decrease.

OG mentions that it doesn't explain the sudden decrease.

However, in my view, is there a sudden decrease at all?

300/day * 5 month season again runs in several thousands.

Can someone explain this!

Posted from my mobile device
GMAT Club Verbal Expert
Joined: 13 Aug 2009
Status: GMAT/GRE/LSAT tutors
Posts: 6936
Own Kudos [?]: 63968 [1]
Given Kudos: 1794
Location: United States (CO)
GMAT 1: 780 Q51 V46
GMAT 2: 800 Q51 V51
GRE 1: Q170 V170

GRE 2: Q170 V170
Send PM
Re: To prevent a newly built dam on the Chiff River from blocking the rout [#permalink]
1
Kudos
Expert Reply
AashishGautam wrote:
I chose D, the reason being that since there has been a decline the population should decrease.

OG mentions that it doesn't explain the sudden decrease.

However, in my view, is there a sudden decrease at all?

300/day * 5 month season again runs in several thousands.

Can someone explain this!

Posted from my mobile device

The passage tells us that before the dam was in place, "several thousand fish a day swam upriver during spawning season."

In the first season after the dam was completed, "only 300 per day made the journey."

If you take into account that several thousand fish each day dwindled to 300 each day, then yes, there is a sudden decrease in fish making the journey upstream.

I hope that helps!
Senior Manager
Senior Manager
Joined: 17 Sep 2016
Posts: 441
Own Kudos [?]: 85 [0]
Given Kudos: 147
Send PM
Re: To prevent a newly built dam on the Chiff River from blocking the rout [#permalink]
dear experts, DavidTutorexamPAL

GMATNinja, GMATNinjaTwo, VeritasKarishma, AnthonyRitz, CJAnish, MartyTargetTestPrep, AndrewN,VeritasPrepBrian
[/url],fiftyoneverbal
I was confused with B,
both other dams with passes had little decreases while Chiff River dam also with pass had significant decrease, that means both other rivers and Chiff River have possess, but have different decreases. so passes are not the reason led to decrease.

most of the posts say B strengthen, while I thought it is a weakness. OA is C, so I must miss something, but I have no idea, genuinely need you help.

thanks in advance
Volunteer Expert
Joined: 16 May 2019
Posts: 3512
Own Kudos [?]: 6899 [1]
Given Kudos: 500
Re: To prevent a newly built dam on the Chiff River from blocking the rout [#permalink]
1
Kudos
Expert Reply
zoezhuyan wrote:
dear experts, DavidTutorexamPAL

GMATNinja, GMATNinjaTwo, VeritasKarishma, AnthonyRitz, CJAnish, MartyTargetTestPrep, AndrewN,VeritasPrepBrian
[/url],fiftyoneverbal
I was confused with B,
both other dams with passes had little decreases while Chiff River dam also with pass had significant decrease, that means both other rivers and Chiff River have possess, but have different decreases. so passes are not the reason led to decrease.

most of the posts say B strengthen, while I thought it is a weakness. OA is C, so I must miss something, but I have no idea, genuinely need you help.

thanks in advance

Hello, zoezhuyan. Be careful in CR questions such as this one to stick to exactly what the argument says. This argument is simple: the fish pass is defective. Why might that be argued? The passage simply tells us that the number of fish per day that swam upriver before the dam/fish pass project was completed versus after has decreased from several thousand to 300. To weaken this argument, we need to look for information that allows for the possibility that this particular fish pass is not defective.

If, as choice (B) states, similar projects on other regional rivers have not led to significant decreases in the number of fish migrating upstream, then there may, in fact, be something problematic with this particular dam and/or fish pass. So, while we cannot eliminate the possibility that the dam is the problem rather than the fish pass, (B) leaves us wondering. It is worth noting that the article the in front of fish pass in the argument might have thrown you off. That is, we are not interested in whether the fish pass is defective in general, but in whether the fish pass on the Chiff River is defective. We cannot look to fish passes on other regional rivers to rule out the possibility that the one in discussion is faulty.

Meanwhile, choice (C) provides an alternative explanation, one that can logically help reconcile the numbers. If toxic river sediments... were carried downstream and the fish swim upstream, then perhaps these fish could not handle the toxins, and their numbers diminished without respect to the fish pass. We still do not know for sure whether the fish pass is defective, but we have a much more compelling reason to doubt that such a contention is necessarily accurate.

I hope that helps. Thank you for thinking to ask me. (I enjoyed the question.)

- Andrew
Manager
Manager
Joined: 23 Jul 2020
Posts: 150
Own Kudos [?]: 27 [0]
Given Kudos: 30
Location: India
Concentration: Entrepreneurship, Marketing
Schools: Ivey '24 (A)
GMAT 1: 700 Q49 V35
Send PM
Re: To prevent a newly built dam on the Chiff River from blocking the rout [#permalink]
Just one thing here. How could one possibly conclude that these toxic substance were life threatening to fish. It could have been toxic for humans but not for fish. Even though I selected C because its more relevant than other option but still it draws no clear infrence and opens itself to some redundancy. To my belief this isnt an absolute weakner.
Volunteer Expert
Joined: 16 May 2019
Posts: 3512
Own Kudos [?]: 6899 [1]
Given Kudos: 500
Re: To prevent a newly built dam on the Chiff River from blocking the rout [#permalink]
1
Kudos
Expert Reply
Mayank221133 wrote:
Just one thing here. How could one possibly conclude that these toxic substance were life threatening to fish. It could have been toxic for humans but not for fish. Even though I selected C because its more relevant than other option but still it draws no clear infrence and opens itself to some redundancy. To my belief this isnt an absolute weakner.

I agree, Mayank2211333, but you have to keep an eye on the question itself:

Quote:
Which of the following, if true, most seriously weakens the argument?

You said yourself that (C) "is more relevant," and if it makes the strongest case of the answers presented, you ought to choose it, plain and simple. In most CR questions, your goal is to find the most reasonable answer of the five, not one that presents an airtight case, and the closer you follow the linear logic of the passage—the less you bring in through outside association—the better off you will find yourself. (I used to overthink CR questions and find the answers somewhat arbitrary, but then I taught myself to follow the rules of GMAT™ logic.)

Good luck with your studies.

- Andrew
Manager
Manager
Joined: 23 Jul 2020
Posts: 150
Own Kudos [?]: 27 [0]
Given Kudos: 30
Location: India
Concentration: Entrepreneurship, Marketing
Schools: Ivey '24 (A)
GMAT 1: 700 Q49 V35
Send PM
Re: To prevent a newly built dam on the Chiff River from blocking the rout [#permalink]
Thanks Andrew. Can you help me with how to be aligned with linear logic? I mean that's like the biggest problem I'm facing with most of the CR question. Can you share those GMAT™ logic rules.
Volunteer Expert
Joined: 16 May 2019
Posts: 3512
Own Kudos [?]: 6899 [0]
Given Kudos: 500
Re: To prevent a newly built dam on the Chiff River from blocking the rout [#permalink]
Expert Reply
Mayank221133 wrote:
Thanks Andrew. Can you help me with how to be aligned with linear logic? I mean that's like the biggest problem I'm facing with most of the CR question. Can you share those GMAT™ logic rules.

Hello again, Mayank221133. I would start with the Ultimate CR Guide for Beginners, by GMATNinja, as well as Mod Nightblade's Quick Guide to CR Proficiency. Both posts have links to other helpful posts. These types of questions take a lot of review time for most people to understand, so when you practice CR questions, look them up in the forum to see which posts cut to the quick and put a finger on exactly why one answer choice stands alone as the correct one. Pay particular attention to what makes other answers incorrect. With consistent practice and a lot of review, you can align your thinking with this linear logic I keep bandying about in no time. Again, just appreciate the process of review. When you spend more time with questions, particularly in the beginning, you will set yourself up for success on subsequent questions; when you place an emphasis on completing more and more questions, you will simply make more and more mistakes, many of which will likely overlap, and you will exhaust valuable study material in the process.

If you come up with further questions along the way, feel free to reach out to the community. We want you to succeed.

- Andrew
Intern
Intern
Joined: 12 Sep 2017
Posts: 28
Own Kudos [?]: 10 [0]
Given Kudos: 16
Location: Korea, Republic of
GMAT 1: 720 Q50 V37
GPA: 3.6
Send PM
Re: To prevent a newly built dam on the Chiff River from blocking the rout [#permalink]
To prevent a newly built dam on the Chiff River from blocking the route of fish migrating to breeding grounds upstream, the dam includes a fish pass, a mechanism designed to allow fish through the dam. Before the construction of the dam and fish pass, several thousand fish a day swam upriver during spawning season. But in the first season after the project's completion, only 300 per day made the journey. Clearly, the fish pass is defective.

P : New build dam include fish pass <- allows fish to pass dam
P : Before the construction several thousand fish a day swam upriver
P : After the construction, only 300 per day do
C : the fish pass is defective

Weaken : Any information that explains why only 300 per day swim upstream besides of fish pass will weaken the conclusion.

Which of the following, if true, most seriously weakens the argument?

A. Fish that have migrated to the upstream breeding grounds do not return down the Chiff River again.
-> This information cannot explain the change that happened before the construction and after the construction. Given the statement was true, it is hard to believe that more than thousands of fishes have swum upward.

B. On other rivers in the region, the construction of dams with fish passes has led to only small decreases in the number of fish migrating upstream.
-> Tihs option only gives reason to belive the particular fish path in the dam is defective contrary to other dam's

C. The construction of the dam stirred up potentially toxic river sediments that were carried downstream.
-> Correct. This option gives another explanation rather than the fish path itself as a reason for the declined migrating fish population.

D. Populations of migratory fish in the Chiff River have been declining slightly over the last 20 years.
-> Irrelevant. The slight decrease cannot explain the discrepancy between thousands and 300.

E. During spawning season, the dam releases sufficient water for migratory fish below the dam to swim upstream.
-> Strengthen. If the fish population migrating upward has decreased even though the dam was releasing sufficient water, the possibility that the fish path is defective rises.
Intern
Intern
Joined: 20 Jul 2020
Posts: 3
Own Kudos [?]: 0 [0]
Given Kudos: 34
Send PM
Re: To prevent a newly built dam on the Chiff River from blocking the rout [#permalink]
[quote="GMATNinja"][quote="NAvinash"]VeritasKarishma, GMATNinja

What if quite a number of fishes migrated and there are not enough fishes left?
(Option A)
Manager
Manager
Joined: 12 Jul 2020
Posts: 82
Own Kudos [?]: 11 [0]
Given Kudos: 109
Location: United Kingdom
GMAT 1: 690 Q49 V34
Send PM
Re: To prevent a newly built dam on the Chiff River from blocking the rout [#permalink]
Unlike everyone stated here, I think C is not the best answer. A is the correct answer.

To weaken the argument (fish pass is defective), we need to find evidence that the fish pass is working.

Option A: it shows that the drop in data (from few thousands fishes to 300) is due to gradual migration of fish upstream and gone, hence the fish trap could well be just working but less fishes use it = Good answer

Option C: similar to A this attempts to address fish population, but this is an inferior answer because:
> it assumes the toxic kills the fish, but the fishes could well be just ill and no change to population which means fish trap is defective.

I won't go into B,D,E as they are obvious.
Intern
Intern
Joined: 08 Dec 2021
Posts: 21
Own Kudos [?]: 0 [0]
Given Kudos: 5
Send PM
Re: To prevent a newly built dam on the Chiff River from blocking the rout [#permalink]
KarishmaB GMATNinja

In option C -> The construction of the dam also includes the 'Fish-pass' right? So why can't we say that the Fish-pass stirred up the toxic material leading to fewer fish swimming upstream?

In option E -> I figured, the dam releases sufficient water for migratory fish below the dam to swim upstream. so even though only 300 fish go through the fish pass, there are some fish that swim upstream below the dam via other way tallying into a total of 1000's. So eventually it weakens the argument that fish-pass is defective.

Please let me where I'm going wrong with this approach.
Tutor
Joined: 16 Oct 2010
Posts: 14891
Own Kudos [?]: 65409 [0]
Given Kudos: 431
Location: Pune, India
Send PM
Re: To prevent a newly built dam on the Chiff River from blocking the rout [#permalink]
Expert Reply
muralis18 wrote:
KarishmaB GMATNinja

In option C -> The construction of the dam also includes the 'Fish-pass' right? So why can't we say that the Fish-pass stirred up the toxic material leading to fewer fish swimming upstream?

In option E -> I figured, the dam releases sufficient water for migratory fish below the dam to swim upstream. so even though only 300 fish go through the fish pass, there are some fish that swim upstream below the dam via other way tallying into a total of 1000's. So eventually it weakens the argument that fish-pass is defective.

Please let me where I'm going wrong with this approach.


Even if we include construction of fish pass in construction of the dam, it does weaken our conclusion that the fish pass is defective.
It is the construction of the dam that released toxic chemicals downstream (that likely caused fish to die) and hence led to fewer fish swimming upriver. The fish pass may not be defective then. The reason for fewer fish could be that there are fewer fish.

I did not understand your logic for (E). We are given that after the dam, only 300 fish swam upriver. We are also given that the dam would have blocked the migration route but for the fish pass. So I am not sure what you mean by 'below the dam and tallying a total of 1000s'
Intern
Intern
Joined: 17 Oct 2020
Posts: 22
Own Kudos [?]: 6 [0]
Given Kudos: 569
Location: India
Send PM
Re: To prevent a newly built dam on the Chiff River from blocking the rout [#permalink]
Hello,
I am having a difficult time understanding why E could not be a valid answer.
The arguments states "several thousand fish a day swam upstream during SPAWNING season." The later part of the argument says only 300 per day made the journey. Clearly, the fish pass is defective.
Now that option E tells us that "During SPAWNING season the dam release ...." . Now, the several thousand number of fishes was recorded during the Spawning season. In the first season after project completion 300 was recorded. My thinking
GMAT Club Bot
Re: To prevent a newly built dam on the Chiff River from blocking the rout [#permalink]
   1   2   3   
Moderators:
GMAT Club Verbal Expert
6936 posts
GMAT Club Verbal Expert
238 posts
CR Forum Moderator
832 posts